Difference between revisions of "2008 AMC 12A Problems/Problem 1"

(New page: ==Problem == What is the reciprocal of <math>\frac{1}{2}+\frac{2}{3}</math>? <math>\textbf{(A)} \frac{6}{7} \qquad \textbf{(B)} \frac{7}{6} \qquad \textbf{(C)} \frac{5}{3} \qquad \textb...)
 
(Removing all content from page)
Line 1: Line 1:
==Problem ==
 
What is the reciprocal of <math>\frac{1}{2}+\frac{2}{3}</math>?
 
  
<math>\textbf{(A)} \frac{6}{7} \qquad \textbf{(B)} \frac{7}{6}  \qquad \textbf{(C)} \frac{5}{3}  \qquad \textbf{(D)}  3  \qquad \textbf{(E)}  \frac{7}{2} </math>
 
 
==Solution==
 
<math>\left(\frac{1}{2}+\frac{2}{3}\right)^{-1}=\left(\frac{3}{6}+\frac{4}{6}\right)^{-1}=\left(\frac{7}{6}\right)^{-1}=\frac{6}{7}\Rightarrow A</math>.
 
 
==See Also==
 

Revision as of 14:31, 17 February 2008